Những câu hỏi liên quan
Diệp Chi Lê
Xem chi tiết
Nguyễn Trần Diệu Linh
28 tháng 4 2018 lúc 9:38

1.

E = \(\dfrac{3}{1.4}\) + \(\dfrac{3}{4.7}\) + \(\dfrac{3}{7.10}\) + \(\dfrac{3}{10.13}\) + \(\dfrac{3}{13.16}\) + \(\dfrac{3}{16.19}\) + \(\dfrac{3}{19.22}\)

E = 1 - \(\dfrac{1}{4}\) + \(\dfrac{1}{4}\) - \(\dfrac{1}{7}\) + \(\dfrac{1}{7}\) - \(\dfrac{1}{10}\) + ... +\(\dfrac{1}{19}\) - \(\dfrac{1}{22}\)

E = 1 - \(\dfrac{1}{22}\)

E = \(\dfrac{21}{22}\)

2.

(x - 4)(x - 5) = 0

TH1:

x - 4 = 0 => x = 4

TH2:

x - 5 = 0 => x = 5

Vậy: x = 4 hoặc x = 5

Bình luận (4)
Yurika Todo
Xem chi tiết
Cheewin
31 tháng 3 2017 lúc 22:17

a) \(4,5:\left[\left(\dfrac{9-10}{6}\right)-\dfrac{9}{5}+\dfrac{12}{5}\right]-\dfrac{1}{7}\)

\(=4,5:\left(\dfrac{-1}{6}-\dfrac{-3}{5}\right)-\dfrac{1}{7}\)

=\(4,5:\left(\dfrac{-5+18}{30}\right)-\dfrac{1}{7}\)

=\(4,5:\dfrac{13}{30}-\dfrac{1}{7}\)=\(\dfrac{135}{13}-\dfrac{1}{7}=\dfrac{932}{91}\)

b) \(\dfrac{13}{3}:\left(\dfrac{1}{4}+\dfrac{5}{4}\right)-\dfrac{20}{3}\)

=\(\dfrac{13}{3}.\dfrac{2}{3}-\dfrac{20}{3}\)=\(\dfrac{26}{9}-\dfrac{20}{3}=\dfrac{26}{9}-\dfrac{60}{9}=\dfrac{-34}{9}\)

c) \(5.\left(\dfrac{1}{1.4}+\dfrac{1}{4.7}+.....+\dfrac{1}{91.94}\right)\)

\(=5.\left[\dfrac{1}{3}\left(1-\dfrac{1}{4}+\dfrac{1}{4}-\dfrac{1}{7}+...+\dfrac{1}{91}-\dfrac{1}{94}\right)\right]\)

\(=5.\left[\dfrac{1}{3}.\left(1-\dfrac{1}{94}\right)\right]\)

=\(5.\left(\dfrac{1}{3}.\dfrac{93}{94}\right)\)

\(=5.\dfrac{31}{94}=\dfrac{155}{94}\)

Chúc bạn học tốt hehe

Bình luận (2)
Lê Phương Linh
Xem chi tiết
Tăng Ngọc Đạt
28 tháng 8 2023 lúc 19:56

\(\dfrac{3}{1\times4}x+\dfrac{3}{4\times7}x+\dfrac{3}{7\times10}x+...+\dfrac{3}{31\times34}x=33\)

\(x\left(\dfrac{3}{1\times4}+\dfrac{3}{4\times7}+\dfrac{3}{7\times10}+...+\dfrac{3}{31\times34}\right)=33\)

\(x\left(\dfrac{1}{1}-\dfrac{1}{4}+\dfrac{1}{4}-\dfrac{1}{7}+\dfrac{1}{7}-\dfrac{1}{10}+...+\dfrac{1}{31}-\dfrac{1}{34}\right)=33\)

\(x\left(1-\dfrac{1}{34}\right)=33\)

\(\dfrac{33}{34}x=33\)

\(x=34\)

Bình luận (0)
Võ Ngọc Phương
28 tháng 8 2023 lúc 19:57

\(\dfrac{3}{1.4}x+\dfrac{3}{4.7}x+\dfrac{3}{7.10}x+...+\dfrac{3}{31.34}x=33\)

\(x.3\left(\dfrac{1}{1.4}+\dfrac{1}{4.7}+\dfrac{1}{7.10}+...+\dfrac{1}{31.34}\right)=33\)

\(x.3.\dfrac{1}{3}\left(1-\dfrac{1}{4}+\dfrac{1}{4}-\dfrac{1}{7}+\dfrac{1}{7}-\dfrac{1}{10}+...+\dfrac{1}{31}-\dfrac{1}{34}\right)=33\)

\(x.\left(1-\dfrac{1}{34}\right)=33\)

\(x.\dfrac{33}{34}=33\)

\(x=33:\dfrac{33}{34}=33.\dfrac{34}{33}\)

\(x=34\)

 

Bình luận (0)
Tiên Nữ Bedee
Xem chi tiết
Akai Haruma
12 tháng 5 2021 lúc 23:34

Lời giải:

\(2A=\frac{4}{1.5}+\frac{6}{5.11}+\frac{8}{11.19}+\frac{10}{19.29}+\frac{12}{29.41}\)

\(=1-\frac{1}{5}+\frac{1}{5}-\frac{1}{11}+\frac{1}{11}-\frac{1}{19}+...+\frac{1}{29}-\frac{1}{41}=1-\frac{1}{41}=\frac{40}{41}\)

\(\Rightarrow A=\frac{20}{21}\)

\(3B=\frac{3}{1.4}+\frac{6}{4.10}+\frac{9}{10.19}+\frac{12}{19.31}=1-\frac{1}{4}+\frac{1}{4}-\frac{1}{10}+\frac{1}{10}-\frac{1}{19}+\frac{1}{19}-\frac{1}{31}\)

\(=1-\frac{1}{31}=\frac{30}{31}\)

\(\Rightarrow B=\frac{10}{31}=\frac{20}{62}<\frac{20}{41}\)

Do đó $A>B$

Bình luận (1)
Tiên Nữ Bedee
Xem chi tiết
OH-YEAH^^
11 tháng 5 2021 lúc 20:46

A.2=4/1.5+6/5.11+...+12/29.41

A.2=1-1/5+1/5-1/11+...+1/29-1/41

A.2=1-1/41

A.2=40/41

A=20/41

B.3=3/1.4+6/4.10+...+12/29.31

B.3=1-1/4+1/4-1/10+...+1/29-1/31

B.3=1-1/31

B.3=30/31

B=10/31

Vì 20/41.10/31 nên A>B

Bình luận (0)
Nguyễn Trí Nghĩa
11 tháng 5 2021 lúc 20:46

\(A=\dfrac{2}{1.5}+\dfrac{3}{5.11}+\dfrac{4}{11.19}+\dfrac{5}{19.29}+\dfrac{6}{29.41}\)

\(\Rightarrow2A=\dfrac{4}{1.5}+\dfrac{6}{5.11}+\dfrac{8}{11.19}+\dfrac{10}{19.29}+\dfrac{12}{29.41}\)

\(\Rightarrow2A=1-\dfrac{1}{5}+\dfrac{1}{5}-\dfrac{1}{11}+\dfrac{1}{11}-\dfrac{1}{19}+\dfrac{1}{19}-\dfrac{1}{29}+\dfrac{1}{29}-\dfrac{1}{41}\)

\(\Rightarrow2A=1-\dfrac{1}{41}=\dfrac{40}{41}\)

\(\Rightarrow A=\dfrac{40}{41}:2=\dfrac{20}{41}\)(1)

\(B=\dfrac{1}{1.4}+\dfrac{2}{4.10}+\dfrac{3}{10.19}+\dfrac{4}{19.31}\)

\(\Rightarrow3B=\dfrac{3}{1.4}+\dfrac{6}{4.10}+\dfrac{9}{10.19}+\dfrac{12}{19.31}\)

\(\Rightarrow3B=\dfrac{1}{1}-\dfrac{1}{4}+\dfrac{1}{4}-\dfrac{1}{10}+\dfrac{1}{10}-\dfrac{1}{19}+\dfrac{1}{19}-\dfrac{1}{31}\)

\(\Rightarrow3B=\dfrac{1}{1}-\dfrac{1}{31}=\dfrac{30}{31}\)

\(\Rightarrow B=\dfrac{30}{31}:3=\dfrac{10}{31}\)

\(\Rightarrow B=\dfrac{2}{2}.\dfrac{10}{31}=\dfrac{20}{62}\)

+)Ta có:\(\dfrac{20}{62}< \dfrac{20}{41}\Rightarrow B< A\)

Hay A>B(ĐPCM)

Chúc bn học tốt

Bình luận (2)

Giải:

\(A=\dfrac{2}{1.5}+\dfrac{3}{5.11}+\dfrac{4}{11.19}+\dfrac{5}{19.29}+\dfrac{6}{29.41}\) 

\(2A=\dfrac{4}{1.5}+\dfrac{6}{5.11}+\dfrac{8}{11.19}+\dfrac{10}{19.29}+\dfrac{12}{29.41}\) 

\(2A=\dfrac{1}{1}-\dfrac{1}{5}+\dfrac{1}{5}-\dfrac{1}{11}+\dfrac{1}{11}-\dfrac{1}{19}+\dfrac{1}{19}-\dfrac{1}{29}+\dfrac{1}{29}-\dfrac{1}{41}\) 

\(2A=\dfrac{1}{1}-\dfrac{1}{41}\) 

\(2A=\dfrac{40}{41}\) 

\(A=\dfrac{40}{41}:2\) 

\(A=\dfrac{20}{41}\) 

\(B=\dfrac{1}{1.4}+\dfrac{2}{4.10}+\dfrac{3}{10.19}+\dfrac{4}{19.31}\) 

\(3B=\dfrac{3}{1.4}+\dfrac{6}{4.10}+\dfrac{9}{10.19}+\dfrac{12}{19.31}\) 

\(3B=\dfrac{1}{1}-\dfrac{1}{4}+\dfrac{1}{4}-\dfrac{1}{10}+\dfrac{1}{10}-\dfrac{1}{19}+\dfrac{1}{19}-\dfrac{1}{31}\) 

\(3B=\dfrac{1}{10}-\dfrac{1}{31}\) 

\(3B=\dfrac{21}{310}\) 

\(B=\dfrac{21}{310}:3\) 

\(B=\dfrac{7}{310}\) 

Vì \(\dfrac{20}{41}>\dfrac{7}{310}\) nên A>B

Bình luận (0)
Nguyễn Văn Phúc Lâm
Xem chi tiết
boi đz
18 tháng 8 2023 lúc 14:35

\(\dfrac{1}{2}-\dfrac{2}{3}+\dfrac{3}{4}-\dfrac{4}{5}+\dfrac{5}{6}-\dfrac{6}{7}-\dfrac{6}{5}+\dfrac{4}{5}-\dfrac{3}{4}+\dfrac{2}{3}-\dfrac{1}{2}\)

\(=\left(\dfrac{1}{2}-\dfrac{1}{2}\right)+\left(-\dfrac{2}{3}+\dfrac{2}{3}\right)+\left(\dfrac{3}{4}-\dfrac{3}{4}\right)+\left(-\dfrac{4}{5}+\dfrac{4}{5}\right)+\left(\dfrac{5}{6}-\dfrac{6}{7}-\dfrac{6}{5}\right)\)

\(=0+0+0+0-\dfrac{257}{210}\)

\(=\dfrac{257}{210}\)

Bình luận (0)
Nguyễn Văn Phúc Lâm
18 tháng 8 2023 lúc 14:32

Có ai biết câu này không, làm giúp mình với

Bình luận (0)
tthinh12
18 tháng 8 2023 lúc 14:40

Cái đó thì chịu.

Bình luận (0)
0o0^^^Nhi^^^0o0
Xem chi tiết
Trần Hà Trang
15 tháng 4 2017 lúc 20:26

A=1/15-1/16+1/16-1/17+...+1/2016-1/2017

A=1/15-1/2017

A=2002/30255

Bình luận (0)
Trần Hà Trang
15 tháng 4 2017 lúc 20:29

C=1/3[3/5.8+3/8.11+...+3/101.104]

C=1/3[1/5-1/8+1/8-1/11+...+1/101-1/104]

C=1/3[1/5-1/104]

C=1/3.99/520

C=33/520

Bình luận (0)
Trần Hà Trang
15 tháng 4 2017 lúc 20:32

D=2/3[3/1.4+3/4.7+.....+3/101.104]

D=2/3[1-1/4+1/4-1/7+.....+1/101-1/104]

D=2/3[1-1/104]

D=2/3.103/104

D=103/156

Bình luận (0)
blabla
Xem chi tiết
Minh Phương
26 tháng 7 2023 lúc 9:25

\(a.\dfrac{-4}{7}-\dfrac{5}{13}\times\dfrac{-39}{25}+\dfrac{-1}{42}:\dfrac{-5}{6}\) 

\(=\dfrac{-4}{7}+\dfrac{3}{5}+\dfrac{1}{35}\) \(=\dfrac{1}{35}+\dfrac{1}{35}=\dfrac{2}{35}\) 

\(b.\dfrac{2}{9}\times\left[\dfrac{4}{5}:\left(\dfrac{1}{5}-\dfrac{2}{15}\right)+1\dfrac{2}{3}\right]-\dfrac{-5}{27}\) 

\(=\dfrac{2}{9}\times\left[\dfrac{4}{5}:\dfrac{1}{15}+\dfrac{5}{3}\right]-\dfrac{-5}{27}\) 

\(=\dfrac{2}{9}\times\left(12+\dfrac{5}{3}\right)-\dfrac{-5}{27}\) 

\(=\dfrac{2}{9}\times\dfrac{41}{3}-\dfrac{-5}{27}=\dfrac{82}{27}-\dfrac{-5}{27}=\dfrac{29}{9}\)

Bình luận (0)
Mai gia bảo
Xem chi tiết
HT.Phong (9A5)
18 tháng 7 2023 lúc 15:43

Bài 4:

\(\dfrac{3}{4}+y:\dfrac{2}{5}=\dfrac{37}{16}\)

\(\Rightarrow y:\dfrac{2}{5}=\dfrac{37}{16}-\dfrac{3}{4}\)

\(\Rightarrow y:\dfrac{2}{5}=\dfrac{25}{16}\)

\(\Rightarrow y=\dfrac{2}{5}\cdot\dfrac{25}{16}\)

\(\Rightarrow y=\dfrac{5}{8}\)

________________

\(456+y:87=23987\)

\(\Rightarrow y:87=23987-456\)

\(\Rightarrow y:87=23531\)

\(\Rightarrow y=23531\cdot87\)

\(\Rightarrow y=2047197\)

Bình luận (0)

a)\(\dfrac{4}{5}\times\dfrac{5}{8}:\dfrac{4}{5}\)

\(=\left(\dfrac{4}{5}:\dfrac{4}{5}\right)\times\dfrac{5}{8}\)

\(=1\times\dfrac{5}{8}=\dfrac{5}{8}\)

b)\(\dfrac{5}{6}+\left(\dfrac{1}{2}:\dfrac{3}{2}+\dfrac{4}{5}\right)\)

\(=\dfrac{5}{6}+\left(\dfrac{1}{3}+\dfrac{4}{5}\right)\)

\(=\dfrac{5}{6}+\dfrac{17}{15}\)

\(=\dfrac{59}{30}\)

Bài 2:

a) \(\dfrac{3}{4}+y:\dfrac{2}{5}=\dfrac{37}{16}\)

\(y:\dfrac{2}{5}=\dfrac{37}{16}-\dfrac{3}{4}\)

\(y:\dfrac{2}{5}=\dfrac{25}{16}\)

\(y=\dfrac{25}{16}\times\dfrac{2}{5}\)

\(y=\dfrac{5}{8}\)

b)\(456+y:87=23987\)

\(y:87=23987-456\)

\(y:87=23531\)

\(y=23531\times87\)

\(y=2047197\)

Bình luận (0)
Nguyễn Minh Dương
18 tháng 7 2023 lúc 15:46

a) 4/5 x 5/8 : 4/5

= 5/8

b) 5/6 + ( 1/2 : 3/2 + 4/5)

= 5/6 + (1/3 + 4/5)

= 5/6 + 17/15

= 59/30

B4:

3/4 + y : 2/5 = 37/16

y : 2/5 = 25/16

y = 5/8.

456 + y : 87 = 23987

y : 87 = 23531

y = 2047197.

Bình luận (0)